***
This Logical Reasoning question is from the December 2000 LSAT.
The question stem asks what would be sufficient to make the argument work. In other words, what would fill the disconnect between the premises and the conclusion?
Breakdown:
Premise 1: Some government economists view their countries as immune to outside influence.
Premise 2: But economies are always open systems – international trade affects prices and wages.
Conclusion: Government economists must therefore look beyond national borders if their nation's economies are to prosper.
There is an obvious hole in this argument. It assumes the economies cannot prosper unless the economists truly understand all influences. (There's no real reason to assume this.)
Something that fills this hole will therefore be sufficient (enough) to make the argument work.
Choice A is correct for this reason because it restates this assumption. If Choice A is assumed, the conclusion follows from the premises.
Choice E is attractive because it's true, according to the stimulus. However, we’re looking for information to make the argument work. Although E is true, it won’t make the conclusion valid unless greater understanding on the part of economists is necessary for a prosperous economy.
wow i did not understand that question at all i for sure thought it was D ... can u explain this one?
ReplyDeleteHere is something that I think is important to note because I've been burned by it more than once. When you see the word "assumed" such as in this question, don't automatically think it's an assumption question. This one is a justify the conclusion questions which is totally different. Initially I chose E) but then I looked at what the question is asking and it's asking for something that must be true for the conclusion to follow not for what is assumed by the stimulus.
ReplyDelete